LSAT and Law School Admissions Forum

Get expert LSAT preparation and law school admissions advice from PowerScore Test Preparation.

 eyeofthetiger
  • Posts: 13
  • Joined: Apr 02, 2014
|
#15098
Hi team,

This one is a doosy for me, mainly because I'm finding it hard to bridge a gap (that I perceive is there). Namely, I am missing modifiers that should be in my conditional reasoning but aren't, and therefore, I don't have anything to attack the answers with. My prephrase is rendered useless.

The stimulus is as follows: "It is wrong for the government to restrict the liberty of individuals, except perhaps in those cases when to fail to do so would allow individuals to cause harm. Yet, to publish something is a liberty, and to offend is not to cause harm."

Here is my set-up based on that:

1) RL (restrict liberty) -> /CH (not cause harm)
2) P (publish) -> L (liberty)
3) O (offend) -> /CH

My next step was to notice that two necessary conditions were the same, so I took the contrapositives of the statements and found that:

CH -> /RL & /0

I don't feel prepared to attack the questions at all with this, and am not sure what I am missing. Any thoughts would be appreciated.
 Robert Carroll
PowerScore Staff
  • PowerScore Staff
  • Posts: 1787
  • Joined: Dec 06, 2013
|
#15109
tiger,

The politician's first sentence establishes the general rule that it is wrong for the government to restrict liberty, with a possible (and only possible) exception where failing to restrict liberty would allow individuals to cause harm. As this exception is only possible, we cannot make a firm inference from a situation where someone might cause harm. We know, however, that, if there is any exception, it can only exist when there is a chance for individuals to cause harm, so if that chance is absent, it is definitely wrong for the government to restrict liberty.

Thus, we can say "If something is part of an individual's liberty AND there is no way for someone to cause harm with that liberty, then it is wrong for the government to restrict that liberty."

Using this on the second sentence, if publishing something is a liberty and causing offense is not a harm, then we know that offense along cannot comprise the chance of causing harm that would constitute a possible exception to the general rule that it is wrong for government to restrict liberty. But we don't know that yet - there are certainly possible unnamed harms that could go along with publishing that the stimulus doesn't discuss, so we can't be certain (and we need to be certain in a Must Be True!) whether restricting liberty in the case of publishing is or isn't wrong.

Look at the answers:

Answer choice (A) tells us it is not right for the government to restrict the publication of literature that is only offensive. "Not right" = "wrong" and the stimulus already told us that publishing is a liberty. We know that restricting liberty is wrong in all cases where there is no chance of causing harm, and the stimulus told us that giving offense is not causing harm. Is there any possible other harm that could be caused? No, because this answer choice says the publication is "only offensive." There is not other possible effect, so no other possible cause of harm to consider.

Answer choice (B) is too absolute; we know there is a possible exception where failing to restrict liberty would allow individuals to cause harm, but that possible exception is ONLY possible. This answer tries to go from the qualified language of the stimulus to absolute language in the answer choice, never a good thing for a Must Be True.

Answer choice (C) misuses "offensive" to describe government behavior, not the publication. It totally misses the mark.

Answer choice (D) fails to discuss whether government action is right or wrong, focusing on an individual criterion of right and wrong that is simply not discussed by the stimulus.

Answer choice (E) adds the word "serious" to state new information - what do we know from the stimulus about how "serious harm" affects things? Nothing, so this goes beyond the information in the stimulus.

Robert Carroll
 eyeofthetiger
  • Posts: 13
  • Joined: Apr 02, 2014
|
#15115
Perfect, thanks Rob!
 jm51
  • Posts: 8
  • Joined: Aug 29, 2014
|
#16252
Hi,

Can someone please explain to me why answer choice (B) is a mistaken negation? In the official answer guide it says that is a secondary reason why that answer is wrong, but I don't see any conditional word indicators.

Thank you!
 Jon Denning
PowerScore Staff
  • PowerScore Staff
  • Posts: 904
  • Joined: Apr 11, 2011
|
#16327
Hey jm,

Thanks for the message and welcome to the Forum!

Answer choice (B) for this one does actually use conditional language, as the word "when" is a sufficient condition indicator (functions similarly to "if"). So if we diagrammed (B), we'd get something like:

..... Harm :arrow: Not Wrong Restrict Liberty

And the reason that's a loose Mistaken Reversal--I wouldn't call it a perfect MR because, as Robert mentions, the first sentence isn't perfectly absolute the way most conditional reasoning is (the word "perhaps" softens it a bit)--is because if we diagrammed the first sentence conditionally we'd show it as:

..... Not Wrong Restrict Liberty :arrow: Harm

where knowing that it was not wrong (or permissible) to actually restrict individual liberty would tell you that doing so must allow those individuals to cause harm. Put another way, if liberty is going to be restricted, you know the one possible reason to restrict it--the exception to freedom: cause harm--must occur.

So in that sense (B) does reverse the ideas from the first sentence, and would generally fall under the category of a Mistaken Reversal.

I hope that helps!

Jon
 jm51
  • Posts: 8
  • Joined: Aug 29, 2014
|
#16386
Hi Jon,

Thank you so much, that is incredibly helpful! :)
 kristinaroz93
  • Posts: 160
  • Joined: Jul 09, 2015
|
#19223
I have looked at both the forum explanation of this site as well as the online student center explanation, but would still like to see if my diagramming of this problem is correct.

The problem starts of by saying, "Politican: It is wrong for the government to restrict the liberty of individuals...."

This is how I diagrammed the problem:
not wrong for gov't to restrict liberty-->individual allowed to cause harm
publish-->liberty
offend--> not cause harm

so if I create a chain inference this is what I get:
offend--> not cause harm-->wrong for gov't to restrict liberty.

Answer choice A, however, which is said to be the correct answer seems to be diagrammed like this:
wrong for gov't to restrict liberty---> offensive
(offensive here is a necessary condition since it is modified by the term only)
And that appears to be a mistaken reversal, which is why I do not understand why this is the correct answer choice.

Where am I going wrong with this problem or what is it that I am missing?
 Nikki Siclunov
PowerScore Staff
  • PowerScore Staff
  • Posts: 1362
  • Joined: Aug 02, 2011
|
#19231
Hi kristinaroz93,

Thanks for your question. You made a mistake diagramming the first statement. The politician states that it is generally wrong for the gov't to restrict liberty, except in those cases when to fail to do so would cause harm. In other words, we should NOT restrict individual liberties EXCEPT in certain specified cases.

Whenever test makers use words such as "unless," "except," or "without," apply the Unless Equation: the term referenced by "unless," "except," or "without" becomes the necessary condition. The remainder must be negated and becomes the sufficient condition.

By this logic, "in those cases when to fail to do so would cause harm" is the necessary condition. The remainder ("NOT restrict liberties") should be negated to become the sufficient condition ("restrict liberties"). In other words, if the government is to restrict individual liberties, it must do so only in those cases where failing to do so would cause harm:

Restrict liberties :arrow: Failure to restrict would cause harm

Onto the next statement: to publish something is a liberty (Publish :arrow: Liberty), and to offend is not to cause harm (Offend :dblline: Harm).

So, let's put these statements together. If publishing is a form of liberty, the conditional relationship in the first statement will apply. Can we restrict publishing? Only when failing to do so would cause harm. But, apparently offending others clearly does not cause harm. So, if all publishing does is offend others, then it doesn't cause harm... and so it shouldn't be restricted. This prephrase matches (A).

Let me know if this makes sense.

Thanks!
 kristinaroz93
  • Posts: 160
  • Joined: Jul 09, 2015
|
#19235
Dear Nikki,

Is this what you are saying the chain inference would look like?

Publish -->liberty-->offend--> not cause harm
not cause harm--> not restrict liberty


Thanks in advance!
 Nikki Siclunov
PowerScore Staff
  • PowerScore Staff
  • Posts: 1362
  • Joined: Aug 02, 2011
|
#19237
Hi kristinaroz93,

I'm afraid not. The following two conditional relationships are independent of each other:

Publish :arrow: Liberty
Offend :arrow: NOT cause harm

As previously discussed, the main conditional relationship is in the first sentence, and it's essentially a principle:

Restrict liberties :arrow: Failure to restrict would cause harm

Since publishing is a form of liberty, this principle will apply to publishing. Simply replace "liberties" with "publishing." So, what necessary condition do you need to meet in order to restrict publishing? You need to show that failure to restrict it would necessarily cause harm.

So, is publishing harmful? If all it does it offend people, which is what answer choice (A) says, then no - publishing would not be harmful, because to offend is not to cause harm. So is it OK to restrict publishing? No, because failure to restrict it will not cause harm. If publishing is only offensive, and to offend is not to cause harm, we arrive at the following conditional chain:

Publishing :arrow: Offend :arrow: NOT Cause Harm :arrow: NOT Restrict

... which justifies answer choice (A).

Hope this makes a bit more sense! :)
  • 1
  • 2
  • 3
  • 4
  • 5
  • 6
  • 8

Get the most out of your LSAT Prep Plus subscription.

Analyze and track your performance with our Testing and Analytics Package.